• Profile
Close

Endometrial cancer: MCQ challenge

M3 India Newsdesk Nov 29, 2023

Endometrial cancer, arising from the uterine lining, is linked to postmenopausal bleeding & hormonal imbalances. Explore its key aspects in this quiz, addressing diagnosis, staging, & treatment decisions for medical professionals, providing valuable insights into clinical scenarios.

Endometrial cancer MCQ

1. A 62-year-old postmenopausal woman presents with postmenopausal bleeding. Endometrial biopsy shows Grade 2 endometrioid adenocarcinoma. Which of the following is the most appropriate next step in management?


2. A 67-year-old woman treated for stage IIIA Grade 2 endometrioid endometrial cancer with surgery and adjuvant pelvic radiation calls with a complaint of vaginal bleeding. Which of the following is the most likely cause?


3. Which of the following patients should be screened for Lynch syndrome?


4. A 28-year-old woman presents with heavy menstrual bleeding unresponsive to medical therapy and is found to have complex hyperplasia with atypia on endometrial biopsy. Which of the following is appropriate definitive surgical management?


5. Which imaging study provides the best assessment of the depth of myometrial invasion in endometrial cancer?

Only Doctors with an M3 India account can read this article. Sign up for free or login with your existing account.
4 reasons why Doctors love M3 India
  • Exclusive Write-ups & Webinars by KOLs

  • Nonloggedininfinity icon
    Daily Quiz by specialty
  • Nonloggedinlock icon
    Paid Market Research Surveys
  • Case discussions, News & Journals' summaries
Sign-up / Log In
x
M3 app logo
Choose easy access to M3 India from your mobile!


M3 instruc arrow
Add M3 India to your Home screen
Tap  Chrome menu  and select "Add to Home screen" to pin the M3 India App to your Home screen
Okay